Thanks to visit codestin.com
Credit goes to www.scribd.com

0% found this document useful (0 votes)
210 views31 pages

Compiled Digest

1) Bernardo Argente and his wife Vicenta applied for life insurance and underwent medical examinations. Vicenta concealed that she had previously been diagnosed and treated for alcoholism and a mental health condition. 2) The insurance company denied the claim after Vicenta passed away, arguing she concealed material health information. The court agreed, finding the concealment invalidated the policy since the insurer would not have approved it had the true facts been disclosed. 3) Concealment of material health information, even without direct questions, can void an insurance policy if the non-disclosed facts would have reasonably affected the insurer's decision to approve the policy.

Uploaded by

Angela Aquino
Copyright
© © All Rights Reserved
We take content rights seriously. If you suspect this is your content, claim it here.
Available Formats
Download as DOCX, PDF, TXT or read online on Scribd
0% found this document useful (0 votes)
210 views31 pages

Compiled Digest

1) Bernardo Argente and his wife Vicenta applied for life insurance and underwent medical examinations. Vicenta concealed that she had previously been diagnosed and treated for alcoholism and a mental health condition. 2) The insurance company denied the claim after Vicenta passed away, arguing she concealed material health information. The court agreed, finding the concealment invalidated the policy since the insurer would not have approved it had the true facts been disclosed. 3) Concealment of material health information, even without direct questions, can void an insurance policy if the non-disclosed facts would have reasonably affected the insurer's decision to approve the policy.

Uploaded by

Angela Aquino
Copyright
© © All Rights Reserved
We take content rights seriously. If you suspect this is your content, claim it here.
Available Formats
Download as DOCX, PDF, TXT or read online on Scribd
You are on page 1/ 31

Argente v West Coast examinations with regard to their health were

G.R. No. L-24899 March 19, 1928 untrue. West Coastrefused to pay the claim and
wrote Argente to the effect that the claim was
Facts: Bernardo Argente signed an application for rejected due to fraud. The trial court held the
joint insurance with his wife in the sum of P2,000. policy null and void, hence this appeal.
The wife, Vicenta de Ocampo, signed for the same.
All the information contained in the applications Issue: WON Argente and Ocampo were guilty of
was furnished the agent by Bernardo Argente. concealment and thereby misled the insurer into
Argente was examined by Dr. Sta. Ana, a medical accepting the risk?
examiner for the West Coast. The result was
recorded in the Medical Examiner's Report, and Held: Yes. Petition dismissed.
with the exception of the signature of Bernardo
Argente, was in the hand-writing of Doctor Sta. Vicenta de Ocampo, in response to the question
Ana. But the information or answers to the asked by the medical examiner, answered no to
questions contained on the face of the Medical "Have you ever consulted a physician for or have
Examiner's Report were furnished the doctor by you ever suffered from any ailment or disease of
Argente. the brain or nervous system?" She also answered
none as to the question whether she consumed
Vicenta de Ocampo, wife of the plaintiff, was alcohol of not.
examined at her residence by the same doctor.
To the question, "What physician or physicians, if
The spouses submitted to West Coast Life an any, not named above, have you consulted or been
amended application, increasing the amount to treated by, within the last five years and for what
P15,000, and asked that the policy be dated May illness or ailment?" she answered "None."
15, 1925. The amended application was
accompanied by the documents entitled "Short But the facts show that she was taken to San
Form Medical Report." In both of these documents Lazaro Hospital, her case was diagnosed by the
appear certain questions and answers. admitting physician as "alcoholism, moreover, she
was diagnosed with "phycho-neurosis."
A temporary policy for P15,000 was issued to
Bernardo Argente and his wife as of May 15, but it Section 25 of the Insurance Code defined
was not delivered until the first quarterly premium concealment as "a neglect to communicate that
on the policy was paid. More than thirty days had which a party knows and ought to communicate."
elapsed since the applicants were examined. Each The court held that the alleged concealment was
of them was required to file a certificate of health not immaterial and insufficient to avoid the policy.
before the policy was delivered. In an action on a life insurance policy where the
evidence conclusively shows that the answers to
Vicenta de Ocampo died of cerebral apoplexy. questions concerning diseases were untrue, the
Argente presented a claim in due form to the West truth of falsity of the answers become the
Coast Life Insurance Co. for the payment of the determining factor. If the true facts been disclosed
sum of P15,000. It was apparently disclosed that by the assured, the insurance would never have
the answers given by the insured in their medical been granted.
strong recommendation for the approval of the
Concealment must, in the absence of inquiries, be insurance application. Then Mondragon received a
not only material, but fraudulent, or the fact must letter from Pacific Life disapproving the insurance
have been intentionally withheld. If no inquiries are application. The letter stated that the said life
made and no fraud or design to conceal enters into insurance application for 20-year endowment plan
the concealment the contract is not avoided. is not available for minors below seven years old,
but Pacific Life can consider the same under the
The assurer is entitled to know every material fact Juvenile Triple Action Plan, and advised that if the
of which the assured has exclusive or peculiar offer is acceptable, the Juvenile Non-Medical
knowledge, as well as all material facts which Declaration be sent to the company.
directly tend to increase the hazard or risk which
are known by the assured, or which ought to be or The non-acceptance of the insurance plan by
are presumed to be known by him. And a Pacific Life was allegedly not communicated by
concealment of such facts vitiates the policy. petitioner Mondragon to private respondent Ngo
Hing. Instead, on May 6, 1957, Mondragon wrote
If the assured has exclusive knowledge of material back Pacific Life again strongly recommending the
facts, he should fully and fairly disclose the same, approval of the 20-year endowment insurance plan
whether he believes them material or not. The to children, pointing out that since the customers
determination of the point whether there has or were asking for such coverage.
has not been a material concealment must rest
largely in all cases upon the exact terms of the Helen Go died of influenza. Ngo Hing sought the
contract. payment of the proceeds of the insurance, but
having failed in his effort, he filed the action for the
recovery before the Court of First Instance of Cebu,
Great Pacific v CA which ruled against him.
G.R. No. L-31845 April 30, 1979
Issues:
Facts: Ngo Hing filed an application with the Great 1. Whether the binding deposit receipt constituted
Pacific for a twenty-year endowment policy in the a temporary contract of the life insurance in
amount of P50,000.00 on the life of his one-year question
old daughter Helen. He supplied the essential data 2. Whether Ngo Hing concealed the state of health
which petitioner Mondragon, the Branch Manager, and physical condition of Helen Go, which
wrote on the form. The latter paid the annual rendered void the policy
premium the sum of P1,077.75 going over to the
Company, but he retained the amount of P1,317.00 Held: (1) No. (2) Yes. Petition dismissed.
as his commission for being a duly authorized
agent of Pacific Life. (1) NO. The receipt was intended to be merely a
provisional insurance contract. Its perfection was
Upon the payment of the insurance premium, the subject to compliance of the following conditions:
binding deposit receipt was issued Ngo Hing. (1) that the company shall be satisfied that the
Likewise, petitioner Mondragon handwrote at the applicant was insurable on standard rates; (2) that
bottom of the back page of the application form his if the company does not accept the application and
offers to issue a policy for a different plan, the Estefania Saturnino vs The Philippine American
insurance contract shall not be binding until the Life
applicant accepts the policy offered; otherwise, the G.R. NO. 16163, February 28, 1963
deposit shall be refunded; and (3) that if the
company disapproves the application, the FACTS: In September 1957, Estefania Saturnino
insurance applied for shall not be in force at any was operated for cancer in which her right breast
time, and the premium paid shall be returned to was removed. She was advised by her surgeon that
the applicant. shes not totally cured because her cancer was
malignant. In November 1957, she applied for an
The receipt is merely an acknowledgment that the insurance policy under Philamlife (Philippine
latter's branch office had received from the American Life Insurance Company). She did not
applicant the insurance premium and had accepted disclose the fact that she was operated nor did she
the application subject for processing by the disclose any medical histories. Philamlife, upon
insurance company. There was still approval or seeing the clean bill of health from Estefania
rejection the same on the basis of whether or not waived its right to have Estefania undergo a
the applicant is "insurable on standard rates." Since medical checkup. In September 1958, Estefania
Pacific Life disapproved the insurance application died of pneumonia secondary to influenza. Her
of respondent Ngo Hing, the binding deposit heirs now seek to enforce the insurance claim.
receipt in question had never become in force at
any time. The binding deposit receipt is conditional ISSUE: Whether or not Saturnino is entitled to the
and does not insure outright. This was held in Lim v insurance claim.
Sun.
HELD: No. The concealment of the fact of the
The deposit paid by private respondent shall have operation is fraudulent. Even if, as argued by the
to be refunded by Pacific Life. heirs, Estefania never knew she was operated for
cancer, there is still fraud in the concealment no
2. YES. Ngo Hing had deliberately concealed the matter what the ailment she was operated for.
state of health of his daughter Helen Go. When he Note also that in order to avoid a policy, it is not
supplied data, he was fully aware that his one-year necessary that actual fraud be established
old daughter is typically a mongoloid child. He otherwise insurance companies will be at the
withheld the fact material to the risk insured. mercy of anyone seeking insurance.

The contract of insurance is one of perfect good In this jurisdiction a concealment, whether
faith uberrima fides meaning good faith, absolute intentional or unintentional, entitles the insurer to
and perfect candor or openness and honesty; the rescind the contract of insurance, concealment
absence of any concealment or demotion, however being defined as negligence to communicate that
slight. which a party knows and ought to communicate.

The concealment entitles the insurer to rescind the Also, the fact that Philamlife waived its right to
contract of insurance. have Estefania undergo a medical examination is
not negligence. Because of Estefanias
concealment, Philamlife considered medical
checkup to be no longer necessary. Had Philamlife Court of Appeals reversed. It found that the failure
been informed of her operation, she would have of Jaime Canilang to disclose previous medical
been made to undergo medical checkup to consultation and treatment constituted material
determine her insurability. information which should have been
communicated to Great Pacific to enable the latter
to make proper inquiries. Hence this petition by
Vda Canilang v CA the widow.
G.R. No. 92492 June 17, 1993
Issue: Won Canilang was guilty of
Facts: Canilang was found to have suffered from misrepresentation
sinus tachycardia then bronchitis after a check-up
from his doctor. The next day, he applied for a Held: Yes. Petition denied.
"non-medical" insurance policy with respondent
Grepalife naming his wife, Thelma Canilang, as his There was a right of the insurance company to
beneficiary. This was to the value of P19,700. rescind the contract if it was proven that the
insured committed fraud in not affirming that he
He died of "congestive heart failure," "anemia," was treated for heart condition and other ailments
and "chronic anemia." The widow filed a claim stipulated.
with Great Pacific which the insurer denied on the
ground that the insured had concealed material Apart from certifying that he didnt suffer from
information from it. such a condition, Canilang also failed to disclose in
the that he had twice consulted a doctor who had
Petitioner then filed a complaint against Great found him to be suffering from "sinus tachycardia"
Pacific for recovery of the insurance proceeds. and "acute bronchitis."
Petitioner testified that she was not aware of any
serious illness suffered by her late husband and her Under the Insurance Code:
husband had died because of a kidney disorder. Sec. 26. A neglect to communicate that which a
The doctor who gave the check up stated that he party knows and ought to communicate, is called a
treated the deceased for sinus tachycardia and concealment.
"acute bronchitis." Sec. 28. Each party to a contract of insurance must
communicate to the other, in good faith, all factors
Great Pacific presented a physician who testified within his knowledge which are material to the
that the deceased's insurance application had been contract and as to which he makes no warranty,
approved on the basis of his medical declaration. and which the other has not the means of
She explained that as a rule, medical examinations ascertaining.
are required only in cases where the applicant has
indicated in his application for insurance coverage The information concealed must be information
that he has previously undergone medical which the concealing party knew and should have
consultation and hospitalization. communicated. The test of materiality of such
The Insurance Commissioner ordered Great Pacific information is contained in Section 31:
to pay P19,700 plus legal interest and P2,000.00 as Sec. 31. Materiality is to be determined not by the
attorney's fees. On appeal by Great Pacific, the event, but solely by the probable and reasonable
influence of the facts upon the party to whom the Indeed, the last medical consultation took place
communication is due, in forming his estimate of just the day before the insurance application was
the disadvantages of the proposed contract, or in filed. In all probability, Jaime Canilang went to visit
making his inquiries. his doctor precisely because of the ailment.

The information which Jaime Canilang failed to Canilang's failure to set out answers to some of the
disclose was material to the ability of Great Pacific questions in the insurance application constituted
to estimate the probable risk he presented as a concealment.
subject of life insurance. Had he disclosed his visits
to his doctor, the diagnosis made and medicines
prescribed by such doctor, in the insurance Sunlife v CA
application, it may be reasonably assumed that G.R. No. 105135 June 22, 1995
Great Pacific would have made further inquiries
and would have probably refused to issue a non- Facts: Robert John B. Bacani procured a life
medical insurance policy. insurance contract for himself from Sunlife. He was
issued a policy for P100,000.00, with double
Materiality relates rather to the "probable and indemnity in case of accidental death. The
reasonable influence of the facts" upon the party designated beneficiary was his mother, Bernarda
to whom the communication should have been Bacani.
made, in assessing the risk involved in making or
omitting to make further inquiries and in accepting The insured died in a plane crash. Respondent
the application for insurance; that "probable and Bernarda Bacani filed a claim with petitioner,
reasonable influence of the facts" concealed must, seeking the benefits of the insurance policy taken
of course, be determined objectively, by the judge by her son. Petitioner conducted an investigation
ultimately. and its findings prompted it to reject the claim.

The Insurance Commissioner had also ruled that Sunlife informed Bacani that the insured did not
the failure of Great Pacific to convey certain disclose material facts relevant to the issuance of
information to the insurer was not "intentional" in the policy, thus rendering the contract of insurance
nature, for the reason that Canilang believed that voidable. A check representing the total premiums
he was suffering from minor ailment like a paid in the amount of P10,172.00 was attached to
common cold. Section 27 stated that: said letter.
Sec. 27. A concealment whether intentional or
unintentional entitles the injured party to rescind a Petitioner claimed that the insured gave false
contract of insurance. statements in his application. The deceased
answered claimed that he consulted a Dr.
The failure to communicate must have been Raymundo of the Chinese General Hospital for
intentional rather than inadvertent. Canilang could cough and flu complications. The other questions
not have been unaware that his heart beat would were answered in the negative.
at times rise to high and alarming levels and that
he had consulted a doctor twice in the two (2) Petitioner discovered that two weeks prior to his
months before applying for non-medical insurance. application for insurance, the insured was
examined and confined at the Lung Center of the Materiality is to be determined not by the event,
Philippines, where he was diagnosed for renal but solely by the probable and reasonable
failure. During his confinement, the deceased was influence of the facts upon the party to whom
subjected to urinalysis tests. communication is due, in forming his estimate of
the disadvantages of the proposed contract or in
Bernarda Bacani and her husband filed an action making his inquiries.
for specific performance against petitioner with the
RTC. The court ruled in favor of the spouses and The terms of the contract are clear. The insured is
ordered Sunlife to pay P100,000.00. specifically required to disclose to the insurer
matters relating to his health.
In ruling for private respondents, the trial court
concluded that the facts concealed by the insured The information which the insured failed to
were made in good faith and under a belief that disclose were material and relevant to the approval
they need not be disclosed. The court also held and issuance of the insurance policy. The matters
that the medial history was irrelevant because it concealed would have definitely affected
wasnt medical insurance. petitioner's action on his application, either by
approving it with the corresponding adjustment for
The Court of Appeals affirmed the decision of the a higher premium or rejecting the same. Moreover,
trial court. The appellate court ruled that petitioner a disclosure may have warranted a medical
cannot avoid its obligation by claiming examination of the insured by petitioner in order
concealment because the cause of death was for it to reasonably assess the risk involved in
unrelated to the facts concealed by the insured. accepting the application.
Petitioner's motion for reconsideration was denied.
Vda. de Canilang v. Court of Appeals- materiality of
Hence, this petition. the information withheld does not depend on the
state of mind of the insured. Neither does it
Issue: WON the insured was guilty of depend on the actual or physical events which
misrepresentation which made the contract void. ensue. Good faith" is no defense in concealment.
The insured's failure to disclose the fact that he
Held: Yes. Petition dismissed. was hospitalized raises grave doubts about his
eligibility. Such concealment was deliberate on his
Section 26 of The Insurance Code required a party part.
to a contract of insurance to communicate to the
other, in good faith, all facts within his knowledge The argument, that petitioner's waiver of the
which are material to the contract and as to which medical examination of the insured debunks the
he makes no warranty, and which the other has no materiality of the facts concealed, is untenable.
means of ascertaining. Saturnino v. Philippine American Life Insurance " . .
A neglect to communicate that which a party . the waiver of a medical examination [in a non-
knows and ought to communicate, is called medical insurance contract] renders even more
concealment. material the information required of the applicant
concerning previous condition of health and
diseases suffered, for such information necessarily
constitutes an important factor which the insurer contrary to his answer in the application form.
takes into consideration in deciding whether to Thus, Julita paid for all the hospitalization
issue the policy or not . . . " expenses.

Anent the finding that the facts concealed had no After Ernani was discharged from the MMC, he was
bearing to the cause of death of the insured, it is attended by a physical therapist at home. Later, he
well settled that the insured need not die of the was admitted at the Chinese General Hospital. Due
disease he had failed to disclose to the insurer. It is to financial difficulties, however, respondent
sufficient that his non-disclosure misled the insurer brought her husband home again. In the morning
in forming his estimates of the risks of the of April 13, 1990, Ernani had fever and was feeling
proposed insurance policy or in making inquiries as very weak. Respondent was constrained to bring
held in Henson. him back to the Chinese General Hospital where he
died on the same day.

Philamcare v CA Julita filed an action for damages and


G.R. No. 125678, March 18 reimbursement of her expenses plus moral
damages attorneys fees against Philamcare and its
FACTS: Ernani Trinos applied for a health care president, Dr. Benito Reverente. The Regional Trial
coverage with Philamcare Health Systems, Inc. To court or Manila rendered judgment in favor of
the question Have you or any of your family Julita. On appeal, the decision of the trial court was
members ever consulted or been treated for high affirmed but deleted all awards for damages and
blood pressure, heart trouble, diabetes, cancer, absolved petitioner Reverente. Hence, this petition
liver disease, asthma or peptic ulcer?, Ernani for review raising the primary argument that a
answered No. Under the agreement, Ernani is health care agreement is not an insurance contract;
entitled to avail of hospitalization benefits and out- hence the incontestability clause under the
patient benefits. The coverage was approved for a Insurance Code does not apply.
period of one year from March 1, 1988 to March 1,
1989. The agreement was however extended ISSUES: (1) Whether or not the health care
yearly until June 1, 1990 which increased the agreement is not an insurance contract; (2)
amount of coverage to a maximum sum of P75,000 Whether or not there is concealment of material
per disability. fact made by Ernani

During the period of said coverage, Ernani suffered HELD: (1)YES. Section2 (1)of the Insurance Code
a heart attack and was confined at the Manila defines a contract of insurance as an agreement
Medical Center (MMC) for one month. While in the whereby one undertakes for a consideration to
hospital, his wife Julita tried to claim the benefits indemnify another against loss, damage, or liability
under the health care agreement. However, the arising from an unknown or contingent event.
Philamcare denied her claim alleging that the
agreement was void because Ernani concealed his Section 3 of the Insurance Code states that any
medical history. Doctors at the MMC allegedly contingent or unknown event, whether past or
discovered at the time of Ernanis confinement that future, which my damnify a person having an
he was hypertensive, diabetic and asthmatic, insurable against him, may be insured against.
Every person has an insurable interest in the life
and health of himself. Being a contract of adhesion, the terms of an
insurance contract are to be construed strictly
Section 10 provides that every person has an against the party which prepared the contract the
insurable interest in the life and health (1) of insurer. By reason of the exclusive control of the
himself, of his spouse and of his children. The insurance company over the terms and
insurable interest of respondents husband in phraseology of the insurance contract, ambiguity
obtaining the health care agreement was his own must be strictly interpreted against the insurer and
health. The health care agreement was in the liberally in favor of the insured, especially to avoid
nature of non-life insurance, which is primarily a forfeiture. This is equally applicable to Health Care
contract of indemnity. Once the member incurs Agreements.
hospital, medical or any other expense arising from
sickness, injury or other stipulated contingent, the
health care provider must pay for the same to the Yu Pang Cheng v. CA
extent agreed upon under the contract. G.R. NO. 12465, MAY 29 1959

(2) NO. The answer assailed by petitioner was in FACTS: On September 1950, Yu Pang Eng
response to the question relating to the medical submitted his application for insurance to an
history of the applicant. This largely depends on insurance company [defendant]. He answered no
opinion rather than fact, especially coming from to questions on his medical history (stomach
respondents husband who was not a medical diseases, dizziness, ulcers, vertigo, cancer, tumors,
doctor. Where matters of opinion or judgment are etc.) as well as to the question of WON he
called for answers made I good faith and without consulted any physician regarding said diseases.
intent to deceive will not avoid a policy even Upon payment of the first premium, the company
though they are untrue. issued to him an insurance policy. On December
1950, he went to St. Lukes for medical treatment
The fraudulent intent on the part of the insured but he died two months later. According to the
must be established to warrant rescission of the death certificate, he died of infiltrating medullary
insurance contract. Concealment as a defense for carcinoma, Grade 4, advanced cardiac and of lesser
the health care provider or insurer to avoid liability curvature, stomach metastases spleen.
is an affirmative defense and the duty to establish
such defense by satisfactory and convincing His brother and beneficiary, Yu Pang Cheng
evidence rests upon the provider or insurer. In any [petitioner], demanded from the insurance
case, with or without the authority to investigate, company the payment of the policy proceeds [10k],
petitioner is liable for claims made under the but his demand was refused so he brought the
contract. Having assumed a responsibility under present action. The insurance companys defense
the agreement, petitioner is bound to answer to was that the insured was guilty of
the extent agreed upon. In the end, the liability of misrepresentation and concealment of material
the health care provider attaches once the member facts in that he gave false and untruthful answers
is hospitalized for the disease or injury covered by to questions asked him in his application; hence,
the agreement or wherever he avails of the the effect is the avoiding of the policy.
covered benefits which he has prepaid.
It appears that the insured entered the Chinese premium payments. However, he defaulted and
General Hospital for medical treatment on January the insurance was cancelled.
1950 [before application for insurance policy],
complaining of dizziness, anemia, abdominal pains He then applied for the reinstatement of his
and tarry stools. His illness history shows that this insurance policy in Nov. of 1951 and tendered the
started a year ago as frequent dizziness. An x-ray amount of premium for the years 1950-1951.
picture of his stomach and the diagnosis was that
he suffered from peptic ulcer, bleeding. He stated that he was as of Nov. 1951 of good
health, and that he had no injuries, ailments or
ISSUE: WON INSURED IS GUILTY OF CONCEALMENT illnesses and had not been sick for any case since
OF MATERIAL FACTS 1948 (his medical check up when he applied for
insurance) and that he had not consulted any
HELD: YES. Concealment is a neglect to physician or practitioner for any case since the date
communicate that which a party knows and ought of such latest medical exam.
to communicate. Whether intentional or not,
concealment entitles the insurer to rescind the However, when Vivencio applied for the
contract. The law requires the insured to reinstatement, he was already sick of a fatal
communicate to the insurer all facts within his disease known as carcinoma of the liver and that 4
knowledge which are material to the contract and days prior to his application for insurance, he
which the other party has not the means of consulted a doctor regarding his condition.
ascertaining. The materiality is determined not by
the event but by the probable and reasonable The reinstatement was approved. Vivencio again
influence of the facts upon the party to whom the failed to pay the premiums for the last quarter of
communication is due. Nov. 1951 and as such, Insular life sent him a
notice canceling the policy.
The insureds negative answers to the questions on
his previous ailments, or his concealment of his Vivencio then died. The beneificiaries instituted
hospitalization deprived the insurance company of the present action to recover from Insular life the
the opportunity to make the necessary inquiry as death benefits of a life insurance policy valued at
to the nature of his past illness so that it may form 2T. Insular refused to pay claiming concealment
its estimate relative to the approval of his on the part of Vivencio.
application. Had the insurance company been given
such opportunity, it would not probably consent to Collado contends that Insular life had waived the
the policy issuance. right to rescine the policy in view of its repeated
acceptance of the overdue premiums for the
second and third years.
Colado v. Insular Life
51 OG (No 12) 6269 Municipal court of Manila found for Collado and
Insular filed an appeal with CFI of Manila. CFI
Facts: Vivencio Collado applied for an insurance rendered judgment in favor of Insular and
contract with Insular life in 1948. His application dismissed Collados complaint.
was approved and he began started making
Issue: Whether or not Insular life was estopped against accident and injuries. She filled up the
and could no longer cancel the contract due to the blank application form given to her and filed the
fact that it accepted the tender of overdue same with the respondent insurance corporation.
payments from Vivencio. In the said application form she gave the date of
her birth as July 11, 1904. On the same date, she
Held: NO. paid the sum of P20.00 representing the premium
for which she was issued the corresponding receipt
It is enormously clear that when the deceased signed by an authorized agent of Manila Bankers.
applied for a reinstatement of his policy in Nov.
1951, he had already been afflicted with the fatal Upon the filing and the payment of the premium,
ailment for a period of about four months. the respondent insurance corporation issued to
Furthermore, in submitting together with his Carmen O. Lapuz its Certificate of Insurance. The
application for reinstatement, a health statement policy was to be effective for a period of 90 days.
to the effect that he was in good health, Vivencio During the effectivity of the certificate of insurance
concealed the material fact that he had consulted a Carmen Lapuz died on a vehicular accident in the
doctor and was then found to be afflicted with the North Diversion Road. On June 7, 1969, petitioner
malady. Regina L. Edillon, a sister of the insured and who
was the named beneficiary in the policy, filed her
The acceptance of Insular life of the overdue claim for the proceeds of the insurance, submitting
premiums did not necessarily deprive it of the right all the necessary papers and other requisites.
to cancel the policy in case of default incurred by
the Insured in the payment of future premiums. However, her claim was denied by the respondent
The case would be different had the insured died at corporation hence her filing of complaint in the
any time after the payment of overdue premiums Court of First Instance of Rizal on August 27, 1969.
but previous to the reinstatement of the policy, for The respondent insurance corporation asserts that
the, Insular, by its acceptance of its overdue since Carmen Lapuz was over 60 years of age the
premiums is deemed to have waived its right to policy in question was null and void because there
rescind the policy. is a provision in the certificate of insurance
excluding its liability to pay claims under the policy
The evidence at hand shows that insofar as the in behalf of persons who are under the age of
payment of the last quarterly premium for 1951 sixteen (16) years of age or over the age of sixty
was concerned, Insular had availed of the right to (60) years.
rescind the policy by notifying the Insured that the
policy had lapsed. The trial court dismissed the complaint. Hence, this
petition.

Edillon vs. Manila Bankers Life Insurance Issue: Whether or not the acceptance by the
Corporation private respondent insurance corporation of the
G.R. No. L-34200September 30, 1982 premium and the issuance of the corresponding
certificate of insurance should be deemed a waiver
Facts: Sometime in April 1969, Carmen O, Lapuz of the exclusionary condition of overage stated in
applied Manila Bankers for insurance coverage the said certificate of insurance
claim but Insular Life denied the application as it
Held: Yes. The age of the insured Carmen 0. Lapuz averred that Felicianos application was attended
was not concealed to the insurance company. Her by fraud. It was later found in court that the
application for insurance coverage which was on a insurance agent and the medical examiner of
printed form furnished by private respondent and Insular Life who assisted Feliciano in signing the
which contained very few items of information application knew that Feliciano was already
clearly indicated her age of the time of filing the suffering from tuberculosis; that they were aware
same to be almost 65 years of age. Despite such of the true medical condition of Feliciano yet they
information which could hardly be overlooked in still made it appear that he was healthy in the
the application form, considering its prominence insurance application form; that Feliciano signed
thereon and its materiality to the coverage applied the application in blank and the agent filled the
for, the respondent insurance corporation received information for him.
her payment of premium and issued the
corresponding certificate of insurance without ISSUE: Whether or not Insular Life can avoid the
question. The accident which resulted in the death insurance policy by reason of the fact that its agent
of the insured, a risk covered by the policy, knowingly and intentionally wrote down the
occurred on May 31, 1969 or FORTY-FIVE (45) DAYS answers in the application differing from those
after the insurance coverage was applied for. There made by Feliciano hence instead of serving the
was sufficient time for the private respondent to interests of his principal, acts in his own or
process the application and to notice that the anothers interest and adversely to that of his
applicant was over 60 years of age and thereby principal.
cancel the policy on that ground if it was minded to
do so. HELD: No. Insular Life must pay the insurance
policy. The weight of authority is that if an agent of
If the private respondent failed to act, it is either the insurer, after obtaining from an applicant for
because it was willing to waive such insurance a correct and truthful answer to
disqualification; or, through the negligence or interrogatories contained in the application for
incompetence of its employees for which it has insurance, without knowledge of the applicant fills
only itself to blame, it simply overlooked such fact. in false answers, either fraudulently or otherwise,
Under the circumstances, the insurance the insurer cannot assert the falsity of such
corporation is already deemed in estoppel. Its answers as a defense to liability on the policy, and
inaction to revoke the policy despite a departure this is true generally without regard to the subject
from the exclusionary condition contained in the matter of the answers or the nature of the agents
said policy constituted a waiver of such condition. duties or limitations on his authority, at least if not
brought to the attention of the applicant.

Insular Life Assurance Co., Ltd. vs Serafin Feliciano The fact that the insured did not read the
G.R. No. 47596 (1941) application which he signed, is not indicative of bad
faith. It has been held that it is not negligence for
FACTS: Evaristo Feliciano was issued an insurance the insured to sign an application without first
policy by Insular Life. In September 1935, he died. reading it if the insurer by its conduct in appointing
His heirs (Serafin Feliciano et al) filed an insurance
the agent influenced the insured to place trust and Assurance Corporation. Asian Crusader asked the
confidence in the agent. following question:

Insular Life Assurance Co., Ltd. vs Serafin Feliciano Has any life insurance company ever refused
G.R. No. 47593 (1943) your application for insurance or for reinstatement
of a lapsed policy or offered you a policy different
FACTS: From the courts decision rendered in the from that applied for? If, so, name company and
case of Insular Life Assurance vs Feliciano (1941), date.
Insular Life filed a motion for reconsideration.
Insular avers that Feliciano is not entitled to the Kwong Nam answered No to the above question.
claim because the insurance policy is void ab initio;
that he connived with the insurance agent and the Kwong Nam was also examined by Asian Crusaders
medical examiner; and that at best, Feliciano is medical examiner to whom he disclosed that he
only entitled to refund or the reimbursement of was once operated and a tumor was removed from
what he has paid in premium. his stomach and such was associated with ulcer of
the stomach.
ISSUE: Whether or not Insular Life is correct.
Kwong Nams application was approved. In May
HELD: Yes. This time, the Supreme Court held that 1963, he died. His widow, Ng Gan Zee, filed an
Insular Lifes contention is correct. When Evaristo insurance claim but Asian Crusader refused her
Feliciano, the applicant for insurance, signed the claim as it insisted that Kwong Nam concealed
application in blank and authorized the soliciting material facts from them when he was applying for
agent and/or medical examiner of Insular to write the insurance; that he misrepresented the fact that
the answers for him, he made them his own agents he was actually denied application by Insular Life
for that purpose, and he was responsible for their when he was renewing his application with them;
acts in that connection. If they falsified the answers that Kwong Nam was actually operated for peptic
for him, he could not evade the responsibility for ulcer.
the falsification. He was not supposed to sign the
application in blank. He knew that the answers to ISSUE: (1) Whether or not Ng Gan Zee can collect
the questions therein contained would be the the insurance claim; (2) Whether or not Asian
basis of the policy, and for that very reason he Crusader was deceived into entering the contract
was required with his signature to vouch for truth or in accepting the risk at the rate of premium
thereof. agreed upon because of insured's representation?

HELD: (1) Yes. Asian Crusader was not able to


Ng Gan Zee vs Asian Crusader Life Assurance prove that Kwong Nams statement that Insular
Corporation Life did not deny his insurance renewal with them
G.R. No. 30685, May 20 1983 is untrue. In fact, evidence showed that in April
1962, Insular Life approved Kwong Nams request
FACTS: In May 1962, Kwong Nam applied for a 20- of reinstatement only with the condition that
year endowment policy with Asian Crusader Life Kwong Nams plan will be lowered from P50,000.00
to P20,000.00 considering his medical history.
It has also been held "that the concealment must,
Kwong Nam did not conceal anything from Asian in the absence of inquiries, be not only material,
Crusader. His statement that his operation, in but fraudulent, or the fact must have been
which a tumor the size of a hens egg was removed intentionally withheld."
from his stomach, was only associated with ulcer
of the stomach and not peptic ulcer can be Fraudulent intent on the part of the insured must
considered as an expression made in good faith of be established to entitle the insurer to rescind the
his belief as to the nature of his ailment and contract. And as correctly observed by the lower
operation. Indeed, such statement must be court, "misrepresentation as a defense of the
presumed to have been made by him without insurer to avoid liability is an 'affirmative' defense.
knowledge of its incorrectness and without any The duty to establish such a defense by satisfactory
deliberate intent on his part to mislead Asian and convincing evidence rests upon the defendant.
Crusader. The evidence before the Court does not clearly and
satisfactorily establish that defense."
While it may be conceded that, from the viewpoint
of a medical expert, the information It bears emphasis that Kwong Nam had informed
communicated was imperfect, the same was the appellant's medical examiner of the tumor. His
nevertheless sufficient to have induced Asian statement that said tumor was "associated with
Crusader to make further inquiries about the ulcer of the stomach" should be construed as an
ailment and operation of Kwong Nam. It has been expression made in good faith of his belief as to the
held that where, upon the face of the application, a nature of his ailment and operation.
question appears to be not answered at all or to be
imperfectly answered, and the insurers issue a While the information communicated was
policy without any further inquiry, they waive the imperfect, the same was sufficient to have induced
imperfection of the answer and render the appellant to make further inquiries about the
omission to answer more fully immaterial. ailment and operation of the insured.

(2) No. Petition dismissed. Section 27 of the Section 32 of Insurance Law:


Insurance Law: Section 32. The right to information of material
Sec. 27. Such party a contract of insurance must facts maybe waived either by the terms of
communicate to the other, in good faith, all facts insurance or by neglect to make inquiries as to such
within his knowledge which are material to the facts where they are distinctly implied in other facts
contract, and which the other has not the means of of which information is communicated.
ascertaining, and as to which he makes no
warranty. Where a question appears to be not answered at
all or to be imperfectly answered, and the insurers
"Concealment exists where the assured had issue a policy without any further inquiry, they
knowledge of a fact material to the risk, and waive the imperfection of the answer and render
honesty, good faith, and fair dealing requires that the omission to answer more fully immaterial.
he should communicate it to the assurer, but he
designedly and intentionally withholds the same." The company or its medical examiner did not make
any further inquiries on such matters from the
hospital before acting on the application for The widow, respondent Medarda V. Leuterio, filed
insurance. The fact of the matter is that the against Grepalife.
defendant was too eager to accept the application
and receive the insured's premium. It would be The trial court rendered a decision in favor of
inequitable now to allow the defendant to avoid respondent widow and against Grepalife. The
liability under the circumstances." Court of Appeals sustained the trial courts
decision.

Great Pacific v CA Issues:


G.R. No. 113899. October 13, 1999 1. Whether the Court of Appeals erred in holding
petitioner liable to DBP as beneficiary in a group
Facts: A contract of group life insurance was life insurance contract from a complaint filed by
executed between petitioner Great Pacific and the widow of the decedent/mortgagor?
Development Bank Grepalife agreed to insure the 2. Whether the Court of Appeals erred in not
lives of eligible housing loan mortgagors of DBP. finding that Dr. Leuterio concealed that he had
Wilfredo Leuterio, a physician and a housing debtor hypertension, which would vitiate the insurance
of DBP, applied for membership in the group life contract?
insurance plan. In an application form, Dr. Leuterio 3. Whether the Court of Appeals erred in holding
answered questions concerning his health Grepalife liable in the amount of eighty six
condition as follows: thousand, two hundred (P86,200.00) pesos without
proof of the actual outstanding mortgage payable
7. Have you ever had, or consulted, a physician by the mortgagor to DBP.
for a heart condition, high blood pressure, cancer,
diabetes, lung, kidney or stomach disorder or any Held:
other physical impairment? 1. NO. Petitioner alleges that the complaint was
8. Are you now, to the best of your knowledge, in instituted by the widow of Dr. Leuterio, not the real
good health? party in interest, hence the trial court acquired no
jurisdiction over the case. It argues that when the
Grepalife issued a coverage to the value of Court of Appeals affirmed the trial courts
P86,200.00 pesos. judgment, Grepalife was held liable to pay the
Dr. Leuterio died due to massive cerebral proceeds of insurance contract in favor of DBP, the
hemorrhage. DBP submitted a death claim to indispensable party who was not joined in the suit.
Grepalife. Grepalife denied the claim alleging that The insured private respondent did not cede to the
Dr. Leuterio was not physically healthy when he mortgagee all his rights or interests in the
applied for an insurance coverage. Grepalife insurance, the policy stating that: In the event of
insisted that Dr. Leuterio did not disclose he had the debtors death before his indebtedness with
been suffering from hypertension, which caused the Creditor [DBP] shall have been fully paid, an
his death. Allegedly, such non-disclosure amount to pay the outstanding indebtedness shall
constituted concealment that justified the denial of first be paid to the creditor and the balance of sum
the claim. assured, if there is any, shall then be paid to the
beneficiary/ies designated by the debtor. When
DBPs claim was denied, it collected the debt from
the mortgagor and took the necessary action of medicines taken by Dr. Leuterio were for
foreclosure on the residential lot of private hypertension, the appellant had not proven nor
respondent. produced any witness who could attest to Dr.
Leuterios medical history.
Gonzales vs. Yek Tong Lin- Insured, being the
person with whom the contract was made, is Appellant insurance company had failed to
primarily the proper person to bring suit thereon. establish that there was concealment made by the
Insured may thus sue, although the policy is taken insured, hence, it cannot refuse payment of the
wholly or in part for the benefit of another person claim.
named or unnamed, and although it is expressly
made payable to another as his interest may The fraudulent intent on the part of the insured
appear or otherwise. Although a policy issued to a must be established to entitle the insurer to
mortgagor is taken out for the benefit of the rescind the contract. Misrepresentation as a
mortgagee and is made payable to him, yet the defense of the insurer to avoid liability is an
mortgagor may sue thereon in his own name, affirmative defense and the duty to establish such
especially where the mortgagees interest is less defense by satisfactory and convincing evidence
than the full amount recoverable under the policy. rests upon the insurer.
Insured may be regarded as the real party in
interest, although he has assigned the policy for 3. NO. A life insurance policy is a valued policy.
the purpose of collection, or has assigned as Unless the interest of a person insured is
collateral security any judgment he may obtain. susceptible of exact pecuniary measurement, the
measure of indemnity under a policy of insurance
And since a policy of insurance upon life or health upon life or health is the sum fixed in the policy.
may pass by transfer, will or succession to any The mortgagor paid the premium according to the
person, whether he has an insurable interest or coverage of his insurance.
not, and such person may recover it whatever the
insured might have recovered,[14] the widow of In the event of the debtors death before his
the decedent Dr. Leuterio may file the suit against indebtedness with the creditor shall have been
the insurer, Grepalife. fully paid, an amount to pay the outstanding
indebtedness shall first be paid to the creditor.
2. NO. The medical findings were not conclusive DBP foreclosed one of the deceased persons lots
because Dr. Mejia did not conduct an autopsy on to satisfy the mortgage. Hence, the insurance
the body of the decedent. The medical certificate proceeds shall inure to the benefit of the heirs of
stated that hypertension was the possible cause the deceased person or his beneficiaries.
of death. Hence, the statement of the physician
was properly considered by the trial court as
hearsay. Tan Chay v West Coast
G.R. No. L-27541 November 21, 1927
Contrary to appellants allegations, there was no
sufficient proof that the insured had suffered from Facts: Tan Chay applied for a life insurance policy
hypertension. Aside from the statement of the of for the sum of P10,000 where he was the sole
insureds widow who was not even sure if the beneficiary. The company approved this. The
policy was issued upon the payment by Tan Ceang
of the first year's premium worth P936. The The word "rescind" has a well defined legal
company agreed to pay the beneficiay the amount meaning, and as applied to contracts, it
of the policy upon the receipt of the proofs of the presupposes the existence of a contract to rescind.
death of the insured while the policy was in force. The rescission relates only to the unfulfilled part,
Without any premium due or unpaid, Ceang died. and not to the entire agreement, making the party
Tan Chay plaintiff submitted the proofs of the rescinding liable on notes executed pursuant to the
death of Tan Ceang with a claim for the payment. contract which matured before the rescission. The
The company refused to pay. rescission is the unmaking of a contract, requiring
the same concurrence of wills as that which made
The company alleged that Tan Ceang obtained the it, and nothing short of this will suffice.
policy by means of deceit to the effect that the
medical certificate had false statements about his After a contract has been broken, whether by an
health. They also claimed that he didnt pay the inability to perform it, or by rescinding against right
premium. The court ruled for Tan Chay and or otherwise, the party not in fault may sue the
commanded the company to pay 10,000 pesos. other for the damages suffered, or, if the parties
can be placed in status quo, he may, should he
Issue: WON Section 47 of the Insurance Code prefer, return what he has received and recover in
applies to this case. a suit value of what he has paid or done. The latter
remedy is termed "rescission."
Held: No. Petition dismissed.
In the instant case, the defendant does not seek to
The plaintiff contends that section 47 of the have the alleged insurance contract rescinded. It
Insurance Act should be applied, and that when so only denies that it ever made any contract of
applied, the company is barred and estopped to insurance on the life of Tan Ceang or that any such
plead the matters alleged in its special defense. a contract ever existed. If the defendant never
That section states: made or entered into the contract in question,
there is no contract to rescind, and, hence, section
Whenever a right to rescind a contract of insurance 47 doesnt apply. As stated, an action to rescind a
is given to the insurer by any provision of this contract is founded upon and presupposes the
chapter, such right must be exercised previous to existence of the contract which is sought to be
the commencement of an action on the contract. rescinded.

The defendant contends that section 47 does not If all of the material matters set forth and alleged
apply to this special defense. If in legal effect in the defendant's special plea are true, there was
defendant's special defense is in the nature of an no valid contract of insurance, for the simple
act to rescind "a contract of insurance," then such reason that the minds of the parties never met and
right must be exercised prior to an action enforce never agreed upon the terms and conditions of the
the contract. contract. If such matters are known to exist by a
preponderance of the evidence, they would
Defendant denied that if ever issued the policy in constitute a valid defense to plaintiff's cause of
question. action. Upon the question as to whether or not
they or are not true, the court couldnt say, but no longer considered in force after the insured has
they were sure that section 47 does not apply to died. The key phrase in the second paragraph of
the allegations made in the answer. Section 48 is for a period of two years. The policy
was issued on November 6, 1973 and the insured
died on April 26, 1975. The policy was thus in force
EMILIO TAN vs. COURT OF APPEALS for a period of only one year and five months.
G.R. No. 48049, 29 June 1989 Considering that the insured died before the two-
year period has lapsed, respondent company is not,
FACTS: Tan Lee Siong, father of herein petitioners, therefore, barred from proving that the policy is
applied for life insurance in the amount of void ab initio by reason of the insureds fraudulent
P80,000.00 with respondent company Philippine concealment or misrepresentation. Moreover,
American Life Insurance Company. Said application respondent company rescinded the contract of
was approved and a corresponding policy was insurance and refunded the premiums paid on
issued effective November 5, 1973, with November 11, 1975, previous to the
petitioners as the beneficiaries. On April 26, 1975, commencement of this action on November 27,
Tan Lee Siong died of hepatoma. Hence, petitioners 1975. WHEREFORE, the petition is hereby DENIED
filed with respondent company their claim for the for lack of merit. The questioned decision of the
proceeds of the life insurance policy. However, the Court of Appeals is AFFIRMED.
insurance company denied the said claim and
rescinded the policy by reason of the alleged
misrepresentation and concealment of material Manila Bankers vs Aban
facts made by the deceased Tan Lee Siong in his G.R. No. 175666 July 29, 2013
application for insurance. The premiums paid on
the policy were thereupon refunded. The Facts: On July 3, 1993, Delia Sotero (Sotero) took
petitioners contend that the respondent company out a life insurance policy from Manila Bankers Life
no longer had the right to rescind the contract of Insurance Corporation (Bankers Life), designating
insurance as rescission must allegedly be done respondent Cresencia P. Aban (Aban), her niece, as
during the lifetime of the insured within two years her beneficiary. Petitioner issued Insurance Policy
and prior to the commencement of action. No. 747411 (the policy), with a face value of P
100,000.00, in Soteros favor on August 30, 1993,
ISSUE: Whether or not the insurance company has after the requisite medical examination and
the right to rescind the contract of insurance payment of the insurance premium. On April 10,
despite the presence of an incontestability clause 1996, when the insurance policy had been in force
for more than two years and seven months, Sotero
HELD: YES. The so-called incontestability clause died. Respondent filed a claim for the insurance
precludes the insurer from raising the defenses of proceeds on July 9, 1996. Petitioner conducted an
false representations or concealment of material investigation into the claim, and came out with the
facts insofar as health and previous diseases are following findings: 1. Sotero did not personally
concerned if the insurance has been in force for at apply for insurance coverage, as she was illiterate;
least two years during the insureds lifetime. The 2. Sotero was sickly since 1990; 3. Sotero did not
phrase during the lifetime found in Section 48 of have the financial capability to pay the insurance
the Insurance Law simply means that the policy is premiums on Insurance Policy No. 747411; 4.
Sotero did not sign the July 3, 1993 application for from the effectivity of a life insurance contract and
insurance; and 5. Respondent was the one who while the insured is alive to discover or prove
filed the insurance application, and x x x designated that the policy is void ab initio or is rescindible by
herself as the beneficiary. For the above reasons, reason of the fraudulent concealment or
petitioner denied respondents claim on April 16, misrepresentation of the insured or his agent. After
1997 and refunded the premiums paid on the the two-year period lapses, or when the insured
policy. dies within the period, the insurer must make good
on the policy, even though the policy was obtained
Issue: Whether or not Manila Bankers is barred by fraud, concealment, or misrepresentation. This
from denying the insurance claims based on fraud is not to say that insurance fraud must be
or concealment. rewarded, but that insurers who recklessly and
indiscriminately solicit and obtain business must be
Held: YES. The incontestability clause is a penalized, for such recklessness and lack of
provision in law that after a policy of life insurance discrimination ultimately work to the detriment of
made payable on the death of the insured shall bona fide takers of insurance and the public in
have been in force during the lifetime of the general.
insured for a period of two (2) years from the date
of its issue or of its last reinstatement, the insurer
cannot prove that the policy is void ab initio or is RCBC v. CA
rescindible by reason of fraudulent concealment or G.R. No. 128833 (1998)
misrepresentation of the insured or his agent.
Facts: GOYU applied for credit facilities and
The purpose of the law is to give protection to the accommodations with RCBC. After due evaluation,
insured or his beneficiary by limiting the rescinding a credit facility in the amount of P30 million was
of the contract of insurance on the ground of initially granted. Upon GOYU's application
fraudulent concealment or misrepresentation to a increased GOYU's credit facility to P50 million, then
period of only two (2) years from the issuance of to P90 million, and finally to P117 million
the policy or its last reinstatement.
As security for its credit facilities with RCBC, GOYU
The insurer is deemed to have the necessary executed two REM and two CM in favor of RCBC,
facilities to discover such fraudulent concealment which were registered with the Registry of Deeds
or misrepresentation within a period of two (2) at. Under each of these four mortgage contracts,
years. It is not fair for the insurer to collect the GOYU committed itself to insure the mortgaged
premiums as long as the insured is still alive, only property with an insurance company approved by
to raise the issue of fraudulent concealment or RCBC, and subsequently, to endorse and deliver
misrepresentation when the insured dies in order the insurance policies to RCBC.
to defeat the right of the beneficiary to recover
under the policy. GOYU obtained in its name a total of 10 insurance
policies from MICO. In February 1992, Alchester
Section 48 serves a noble purpose, as it regulates Insurance Agency, Inc., the insurance agent where
the actions of both the insurer and the insured. GOYU obtained the Malayan insurance policies,
Under the provision, an insurer is given two years
issued nine endorsements in favor of RCBC contemporaneous acts, must be given due
seemingly upon instructions of GOYU consideration in order to better serve the interest
of justice and equity.
On April 27, 1992, one of GOYU's factory buildings
in Valenzuela was gutted by fire. Consequently, It is to be noted that 9 endorsement documents
GOYU submitted its claim for indemnity. were prepared by Alchester in favor of RCBC. The
Court is in a quandary how Alchester could arrive
MICO denied the claim on the ground that the at the idea of endorsing any specific insurance
insurance policies were either attached pursuant to policy in favor of any particular beneficiary or
writs of attachments/garnishments issued by payee other than the insured had not such named
various courts or that the insurance proceeds were payee or beneficiary been specifically disclosed by
also claimed by other creditors of GOYU alleging the insured itself. It is also significant that GOYU
better rights to the proceeds than the insured. voluntarily and purposely took the insurance
policies from MICO, a sister company of RCBC, and
GOYU filed a complaint for specific performance not just from any other insurance company.
and damages. RCBC, one of GOYU's creditors, also Alchester would not have found out that the
filed with MICO its formal claim over the proceeds subject pieces of property were mortgaged to
of the insurance policies, but said claims were also RCBC had not such information been voluntarily
denied for the same reasons that AGCO denied disclosed by GOYU itself. Had it not been for GOYU,
GOYU's claims. Alchester would not have known of GOYU's
intention of obtaining insurance coverage in
However, because the endorsements do not bear compliance with its undertaking in the mortgage
the signature of any officer of GOYU, the trial contracts with RCBC, and verify, Alchester would
court, as well as the Court of Appeals, concluded not have endorsed the policies to RCBC had it not
that the endorsements are defective and held that been so directed by GOYU.
RCBC has no right over the insurance proceeds.
On equitable principles, particularly on the ground
Issue: Whether or not RCBC has a right over the of estoppel, the Court is constrained to rule in
insurance proceeds. favor of mortgagor RCBC. RCBC, in good faith,
relied upon the endorsement documents sent to it
Held: YES, RCBC has a right over the insurance as this was only pursuant to the stipulation in the
proceeds. mortgage contracts. We find such reliance to be
It is settled that a mortgagor and a mortgagee have justified under the circumstances of the case.
separate and distinct insurable interests in the GOYU failed to seasonably repudiate the authority
same mortgaged property, such that each one of of the person or persons who prepared such
them may insure the same property for his own endorsements. Over and above this, GOYU
sole benefit. There is no question that GOYU could continued, in the meantime, to enjoy the benefits
insure the mortgaged property for its own of the credit facilities extended to it by RCBC. After
exclusive benefit. In the present case, although it the occurrence of the loss insured against, it was
appears that GOYU obtained the subject insurance too late for GOYU to disown the endorsements for
policies naming itself as the sole payee, the any imagined or contrived lack of authority of
intentions of the parties as shown by their Alchester to prepare and issue said endorsements.
If there had not been actually an implied properly covered against any loss by an
ratification of said endorsements by virtue of insurance company acceptable to RCBC.
GOYU's inaction in this case, GOYU is at the very 2. GOYU voluntarily procured insurance policies to
least estopped from assailing their operative cover the mortgaged property from MICO, no
effects. less than a sister company of RCBC and
definitely an acceptable insurance company to
To permit GOYU to capitalize on its non- RCBC.
confirmation of these endorsements while it 3. Endorsement documents were prepared by
continued to enjoy the benefits of the credit MICO's underwriter, Alchester Insurance
facilities of RCBC which believed in good faith that Agency, Inc., and copies thereof were sent to
there was due endorsement pursuant to their GOYU, MICO and RCBC. GOYU did not assail,
mortgage contracts, is to countenance grave until of late, the validity of said endorsements.
contravention of public policy, fair dealing, good 4. GOYU continued until the occurrence of the
faith, and justice. Such an unjust situation, the fire, to enjoy the benefits of the credit facilities
Court cannot sanction. Under the peculiar extended by RCBC which was conditioned upon
circumstances obtaining in this case, the Court is the endorsement of the insurance policies to be
bound to recognize RCBC's right to the proceeds of taken by GOYU to cover the mortgaged
the insurance policies if not for the actual properties.
endorsement of the policies, at least on the basis
of the equitable principle of estoppel. This Court cannot over stress the fact that upon
receiving its copies of the endorsement documents
GOYU cannot seek relief under Section 53 of the prepared by Alchester, GOYU, despite the absence
Insurance Code which provides that the proceeds written conformity thereto, obviously considered
of insurance shall exclusively apply to the interest said endorsement to be sufficient compliance with
of the person in whose name or for whose benefit its obligation under the mortgage contracts since
it is made. The peculiarity of the circumstances RCBC accordingly continued to extend the benefits
obtaining in the instant case presents a justification of its credit facilities and GOYU continued to
to take exception to the strict application of said benefit therefrom. Just as plain too is the intention
provision, it having been sufficiently established of the parties to constitute RCBC as the beneficiary
that it was the intention of the parties to designate of the various insurance policies obtained by
RCBC as the party for whose benefit the insurance GOYU. The intention of the parties will have to be
policies were taken out. Consider thus the given full force and effect in this particular case.
following: The insurance proceeds may, therefore, be
1. It is undisputed that the insured pieces of exclusively applied to RCBC, which under the
property were the subject of mortgage factual circumstances of the case, is truly the
contracts entered into between RCBC and person or entity for whose benefit the policies
GOYU in consideration of and for securing were clearly intended.
GOYU's credit facilities from RCBC. The
mortgage contracts contained common
provisions whereby GOYU, as mortgagor, Tang v. CA
undertook to have the mortgaged property G.R. No. 48563, MAY 25, 1979
Facts: On Sept. 25, 2965, Lee Su Guat, widow, 61 devolves on the party seeking to enforce it. Here,
years old and illiterate who spoke only Chinese, the insurance company is NOT seeking to enforce
applied for life insurance for 60T with Philamlife. the contract; on the contrary, it is seeking to avoid
The application was in two parts, both in English. its performance.

The second part dealt with her state of health. Her It is petitioner who is seeking to enforce it, even as
answers having shown that she was health, fraud or mistake is NOT alleged. Accordingly,
Philamlife issued her a policy effective Oct. 23, Philamlife was under no obligation to prove that
1965 with her nephew Vicente Tang as beneficiary. the terms of the insurance contract were fully
explained to the other party. Even if we were to
On Nov. 15, 1965, Lee again applied for additional say that the insurer is the one seeking the
insurance of her life for 40T. Since it was only performance of the cont contracts by avoiding
recent from the time she first applied, no further paying the claim, it has to be noted as above stated
medical exam was made but she accomplished Part that there has been NO imputation of mistake of
1 (which certified the truthfulness of statements fraud by the illiterate insured whose personality is
made in Part. 2) represented by her beneficiary. In sum, Art. 1332
is inapplicable, and considering the findings of both
The policy was again approved. On Apri 20 1966, the trial court and the CA as to the Concealment of
Lee Su Guat died of Lung cancer. Lee, the SC affirms their decisions.

Tang claimed the amount o 100T but Philamlife Concurring: J., Antonio
refused to pay on the ground that the insured was In a contract of insurance, each party must
guilty of concealment and misrepresentation. communicate to the other, in good faith, all facts
within his knowledge which are material to the
Both trial court and CA ruled that Lee was guilty of contract, and which the other has no means of
concealment. ascertaining. As a general rule, the failure by the
insured to disclose conditions affecting the risk of
Tangs position, however, is that because Lee was which he is aware makes the contract voidable at
illiterate and spoke only Chinese, she could not be the option of the insurer.
held guilty of concealment of her health history
because the application for insurance was English, The reason for this rule is that insurance policies
and the insurer has not proven that the terms are traditionally contracts uberrimae fidei, which
thereof had been fully explained to her as provided means most abundant good faith, absolute and
by Art. 1332 of CC. perfect candor or openness and honesty,
absence of any concealment or deception
Issue: Whether or not Art. 1332 applies. however slight. Here the CA found that the
insured deliberately concealed material facts about
Held: NO. Art. 1332 is NOT applicable. Under said her physical condition and history and/or
article, the obligation to show that the terms of the concealed with whoever assisted her in relaying
contract had been fully explained to the party who false information to the medical examiner.
is unable to read or understand the language of the Certainly, the petitioner cannot assume
contract, when fraud or mistake is alleged, inconsistent positions by attempting to enforce the
contract of insurance for the purpose of collecting case the amount herein acknowledged shall be
the proceeds of the policy and at the same time returned.
nullify the contract by claiming that it was executed
through fraud or mistake. Issue: WON there was a perfected contract of
insurance
NOTE: Art. 1332: When one of the parties is
unable to read or if the contract is in a language Held: No. Petition dismissed.
not understood by him, and mistake or fraud is
alleged, the person enforcing the contract must The policy for four months is expressly made
show that the terms thereof have been fully subjected to the affirmative condition that "the
explained to him. company shall confirm this agreement by issuing a
policy on said application when the same shall be
submitted to the head office in Montreal."
Lim v Sunlife
G.R. No. L-15774 November 29, 1920 Should the company not issue such a policy, then
this agreement shall be null and void ab initio, and
Facts: Luis Lim of Zamboanga applied for a Sun Life the company shall be held not to have been on the
policy for Php 5,000. He designated his wife, Pilar, risk." This means that the agreement should not go
as beneficiary. The first premium of P433 was paid into effect until the home office of the company
by Lim, then the company issued a "provisional should confirm it by issuing a policy. The
policy." Lim died after the issuance of the provisional policy amounts to nothing but an
provisional policy but before approval of the acknowledgment on behalf of the company, that it
application. has received from the person named therein the
sum of money agreed upon as the first year's
Pilar brought an action to recover from Sun Life the premium upon a policy to be issued upon the
sum of P5,000, the amount named in the application, if the application is accepted by the
provisional policy. She lost in the trial court hence company.
this appeal.
There can be no contract of insurance unless the
The "provisional policy" reads as follows: minds of the parties have met in agreement. In this
The above-mentioned life is to be assured in case, the contract of insurance was not
accordance with the terms and conditions consummated by the parties.
contained or inserted by the Company in the policy
which may be granted by it in this particular case The general rule concerning the agent's receipt
for four months only from the date of the pending approval or issuance of policy is in several
application, provided that the Company shall points, according to Joyce:
confirm this agreement by issuing a policy on said Where an agreement is made between the
application when the same shall be submitted to applicant and the agent whether by signing an
the Head Office in Montreal. Should the Company application containing such condition, or otherwise,
not issue such a policy, then this agreement shall be that no liability shall attach until the principal
null and void ab initio, and the Company shall be approves the risk and a receipt is given buy the
held not to have been on the risk at all, but in such agent, such acceptance is merely conditional, and it
subordinated to the act of the company in secretary at the head office of the company and
approving or rejecting; so in life insurance a issued." The premium of 433 must be returned.
"binding slip" or "binding receipt" does not insure
of itself.
JAMES STOKES vs. MALAYAN
The court held that this second point applied to the G.R. No. L-34768, 24 February 1984
case.
FACTS: Daniel Adolfson had a subsisting Malayan
American jurisprudence tells us of such examples: car insurance policy with coverage against own
damage as well as 3rd party liability when his car
Steinle vs. New York Life Insurance Co.- the figured in a vehicular accident with another car,
amount of the first premium had been paid to an resulting to damage to both vehicles. At the time of
insurance agent and a receipt was given. The paper the accident, Adolfsons car was being driven by
declared that if the application was accepted by James Stokes, who was authorized to do so by
the company, the insurance shall take effect from Adolfson. Stokes, an Irish tourist who had been in
the date of the application but that if the the Philippines for only 90 days, had a valid and
application was not accepted, the money shall be subsisting Irish drivers license but without a
returned. The court held that there was no Philippine drivers license. Adolfson filed a claim
perfection of the contract. with Malayan but the latter refused to pay
contending that Stokes was not an authorized
Cooksey vs. Mutual Life Insurance Co.- the person driver under the Authorized Driver clause of the
applying for the life insurance paid and amount insurance policy in relation to Section 21 of the
equal to the first premium, but the application and Land Transportation Office.
the receipt for the money paid, stipulated that the
insurance was to become effective only when the ISSUE: Whether or not Malayan is liable to pay the
application was approved and the policy issued. insurance claim of Adolfson
There was also no perfection.
HELD: NO. A contract of insurance is a contract of
A binding receipt is a custom where temporary indemnity upon the terms and conditions specified
insurance pending the consideration of the therein. When the insurer is called upon to pay in
application was given until the policy be issued or case of loss or damage, he has the right to insist
the application rejected, and such contracts are upon compliance with the terms of the contract. If
upheld and enforced when the applicant dies the insured cannot bring himself within the terms
before the issuance of a policy or final rejection of and conditions of the contract, he is not entitled as
the application. a rule to recover for the loss or damage suffered.
For the terms of the contract constitute the
However, there was no perfected contract because measure of the insurers liability, and compliance
of the clause in the application and the receipt therewith is a condition precedent to the right of
stipulate expressly that the insurance shall become recovery. At the time of the accident, Stokes had
effective only when the "application shall be been in the Philippines for more than 90 days.
approved and the policy duly signed by the Hence, under the law, he could not drive a motor
vehicle without a Philippine drivers license. He was
therefore not an authorized driver under the refused to make such reimbursement
terms of the insurance policy in question, and notwithstanding the fact that her claim was within
Malayan was right in denying the claim of the its contractual liability under the insurance policy.
insured. Acceptance of premium within the
stipulated period for payment thereof, including The decision of the CA affirmed in toto the decision
the agreed period of grace, merely assures of the RTC of Cavite, the dispositive portion of
continued effectivity of the insurance policy in which states:
accordance with its terms. Such acceptance does
not estop the insurer from interposing any valid IN VIEW OF THE FOREGOING, judgment is hereby
defense under the terms of the insurance policy. rendered ordering defendant PCSI to pay plaintiff
The principle of estoppel is an equitable principle Cayas the sum of P50,000.00 under its maximum
rooted upon natural justice which prevents a liability as provided for in the insurance policy;
person from going back on his own acts and
representations to the prejudice of another whom In this petition for review on certiorari, petitioner
he has led to rely upon them. The principle does seeks to limit its liability only to the payment made
not apply to the instant case. In accepting the by private respondent to Perea and only up to the
premium payment of the insured, Malayan was not amount of P12,000.00. It altogether denies liability
guilty of any inequitable act or representation. for the payments made by private respondents to
There is nothing inconsistent between acceptance the other 3 injured passengers totaling P12,000.00.
of premium due under an insurance policy and the
enforcement of its terms. WHEREFORE, the ISSUE: How much should PCSI pay?
appealed judgment is reversed. The complaint is
dismissed. Costs against appellees. HELD: The decision of the CA is modified, petitioner
only to pay Cayas P12,000,000.00

PERLA COMPANIA DE SEGUROS, INC vs. CA and


CAYAS
G.R. No. 78860, May 28, 1990 The insurance policy provides:
5. No admission, offer, promise or payment shall be
FACTS: Cayas was the registered owner of a Mazda made by or on behalf of the insured without the
bus which was insured with petitioner PERLA written consent of the Company
COMPANIA DE SEGUROS, INC (PCSI). The bus
figured in an accident in Cavite, injuring several of It being specifically required that petitioners
its passengers. One of them, Perea, sued Cayas for written consent be first secured before any
damages in the CFI, while three others agreed to a payment in settlement of any claim could be made,
settlement of P4,000.00 each with Cayas. private respondent is precluded from seeking
reimbursement of the payments made to the other
After trial, the court rendered a decision in favor of 3 victims in view of her failure to comply with the
Perea, Cayas ordered to compensate the latter condition contained in the insurance policy.
with damages. Cayas filed a complaint with the CFI,
seeking reimbursement from PCSI for the amounts
she paid to ALL victims, alleging that the latter
Also, the insurance policy involved explicitly limits A fire broke out on the premises destroying the
petitioners liability to P12,000.00 per person and goods contained in the building. The bank sent a
to P50,000.00 per accident letter of demand to Oriental for indemnity. The
company wasnt ready to give since it was awaiting
Clearly, the fundamental principle that contracts the adjusters report.
are respected as the law between the contracting
parties finds application in the present case. Thus, The company then made an excuse that the
it was error on the part of the trial and appellate insured had not filed any claim with it, nor
courts to have disregarded the stipulations of the submitted proof of loss which is a clear violation of
parties and to have substituted their own Policy Condition No.11, as a result, determination
interpretation of the insurance policy. of the liability of private respondent could not be
made.
We observe that although Cayas was able to prove
a total loss of only P44,000.00, petitioner was Pacific Banking filed in the trial court an action for a
made liable for the amount of P50,000.00, the sum of money for P61,000.00 against Oriental
maximum liability per accident stipulated in the Assurance.
policy. This is patent error. An insurance indemnity,
being merely an assistance or restitution insofar as At the trial, petitioner presented communications
can be fairly ascertained, cannot be availed of by of the insurance adjuster to Asian Surety revealing
any accident victim or claimant as an instrument of undeclared co-insurances with the following:
enrichment by reason of an accident. P30,000 with Wellington Insurance; P25,000 with
Empire Surety and P250,000 with Asian Surety
undertaken by insured Paramount on the same
Pacific v CA property covered by its policy with Oriental
G.R. No. L-41014 November 28, 1988 whereas the only co-insurances declared in the
subject policy are those of P30,000.00 with
Facts: An open fire insurance policy, was issued to Malayan P50,000.00 with South Sea and
Paramount Shirt Manufacturing by Oriental P25.000.00 with Victory.
Assurance Corporation to indemnify P61,000.00,
caused by fire to the factorys stocks, materials and The defense of fraud, in the form of non-
supplies. declaration of co-insurances which was not
pleaded in the answer, was also not pleaded in the
The insured was a debtor of Pacific Banking in the Motion to Dismiss.
amount of (P800,000.00) and the goods described
in the policy were held in trust by the insured for The trial court denied the respondents motion.
Pacific Banking under trust receipts. Oriental filed another motion to include additional
The policy was endorsed to Pacific Banking as evidence of the co-insurance which could amount
mortgagee/ trustor of the properties insured, with to fraud.
the knowledge and consent of private respondent
to the effect that "loss if any under this policy is The trial court still made Oriental liable for P
payable to the Pacific Banking Corporation". 61,000. The CA reversed the trial court decision.
Pacific Banking filed a motion for reconsideration
of the said decision of the respondent Court of Petitioner points out that Condition No. 3 in the
Appeals, but this was denied for lack of merit. policy in relation to the "other insurance clause"
supposedly to have been violated, cannot certainly
Issues: defeat the right of the petitioner to recover the
1. WON unrevealed co-insurances Violated policy insurance as mortgagee/assignee. Hence, they
conditions No. 3 claimed that the purpose for which the
2. WON the insured failed to file the required proof endorsement or assignment was made was to
of loss prior to court action. protect the mortgagee/assignee against any
untoward act or omission of the insured. It would
Held: Yes. Petition dismissed. be absurd to hold that petitioner is barred from
recovering the insurance on account of the alleged
1. YES. Policy Condition No. 3 explicitly provides: violation committed by the insured.
3. The Insured shall give notice to the Company of
any insurance already effected, or which may It is obvious that petitioner has missed all together
subsequently be effected, covering any of the the import of subject mortgage clause which
property hereby insured, and unless such notice be specifically provides:
given and the particulars of such insurance or Loss, if any, under this policy, shall be payable to
insurances be stated in or endorsed on this Policy the PACIFIC BANKING CORPORATION Manila
by or on behalf of the Company before the mortgagee/trustor as its interest may appear, it
occurrence of any loss or damage, all benefit under being hereby understood and agreed that this
this policy shall be forfeited. insurance as to the interest of the
mortgagee/trustor only herein, shall not be
The insured failed to reveal before the loss three invalidated by any act or neglectexcept fraud or
other insurances. Had the insurer known that there misrepresentation, or arsonof the mortgagor or
were many co-insurances, it could have hesitated owner/trustee of the property insured; provided,
or plainly desisted from entering into such that in case the mortgagor or owner/ trustee
contract. Hence, the insured was guilty of clear neglects or refuses to pay any premium, the
fraud. mortgagee/ trustor shall, on demand pay the
same.
Concrete evidence of fraud or false declaration by
the insured was furnished by the petitioner itself The paragraph clearly states the exceptions to the
when the facts alleged in the policy under clauses general rule that insurance as to the interest of the
"Co-Insurances Declared" and "Other Insurance mortgagee, cannot be invalidated; namely: fraud,
Clause" are materially different from the actual or misrepresentation or arson. Concealment of the
number of co-insurances taken over the subject aforecited co-insurances can easily be fraud, or in
property. the very least, misrepresentation.

As the insurance policy against fire expressly Undoubtedly, it is but fair and just that where the
required that notice should be given by the insured insured who is primarily entitled to receive the
of other insurance upon the same property, the proceeds of the policy has by its fraud and/or
total absence of such notice nullifies the policy. misrepresentation, forfeited said right.
Petitioner further stressed that fraud which was It appearing that insured has violated or failed to
not pleaded as a defense in private respondent's perform the conditions under No. 3 and 11 of the
answer or motion to dismiss, should be deemed to contract, and such violation or want of
have been waived. It will be noted that the fact of performance has not been waived by the insurer,
fraud was tried by express or at least implied the insured cannot recover, much less the herein
consent of the parties. Petitioner did not only petitioner.
object to the introduction of evidence but on the
contrary, presented the very evidence that proved
its existence. Perez v CA
G.R. No. 112329. January 28, 2000
2. YES. Generally, the cause of action on the policy
accrues when the loss occurs, But when the policy Facts: Primitivo B. Perez had been insured with the
provides that no action shall be brought unless the BF Lifeman Insurance Corporation for P20,000.00.
claim is first presented extrajudicially in the Sometime in October 1987, an agent of the
manner provided in the policy, the cause of action insurance corporation, visited Perez in Quezon and
will accrue from the time the insurer finally rejects convinced him to apply for additional insurance
the claim for payment coverage of P50,000.00. Virginia A. Perez,
Primitivos wife, paid P2,075.00 to the agent. The
In the case at bar, policy condition No. 11 receipt issued indicated the amount received was a
specifically provides that the insured shall on the "deposit." Unfortunately, the agent lost the
happening of any loss or damage give notice to the application form accomplished by Perez and he
company and shall within fifteen (15) days after asked the latter to fill up another application form.
such loss or damage deliver to the private The agent sent the application for additional
respondent (a) a claim in writing giving particular insurance of Perez to the Quezon office. Such was
account as to the articles or goods destroyed and supposed to forwarded to the Manila office.
the amount of the loss or damage and (b)
particulars of all other insurances, if any. Perez drowned. His application papers for the
Twenty-four days after the fire did petitioner additional insurance of P50,000.00 were still with
merely wrote letters to private respondent to serve the Quezon. It was only after some time that the
as a notice of loss. It didnt even furnish other papers were brought to Manila. Without knowing
documents. Instead, petitioner shifted upon that Perez died, BF Lifeman Insurance Corporation
private respondent the burden of fishing out the approved the application and issued the
necessary information to ascertain the particular corresponding policy for the P50,000.00.
account of the articles destroyed by fire as well as
the amount of loss. Since the required claim by Petitioner Virginia Perez went to Manila to claim
insured, together with the preliminary submittal of the benefits under the insurance policies of the
relevant documents had not been complied with, it deceased. She was paid P40,000.00 under the first
follows that private respondent could not be insurance policy for P20,000.00 but the insurance
deemed to have finally rejected petitioner's claim company refused to pay the claim under the
and therefore there was no cause of action. additional policy coverage of P50,000.00, the
proceeds of which amount to P150,000.00.
The insurance company maintained that the corporation was further conditioned with the
insurance for P50,000.00 had not been perfected following requisites stated in the application form:
at the time of the death of Primitivo Perez. "there shall be no contract of insurance unless and
Consequently, the insurance company refunded until a policy is issued on this application and that
the amount paid. the said policy shall not take effect until the
premium has been paid and the policy delivered to
BF Lifeman Insurance Corporation filed a complaint and accepted by me/us in person while I/We,
against Virginia Perez seeking the rescission and am/are in good health."
declaration of nullity of the insurance contract in
question. BF Lifeman didnt give its assent when it merely
received the application form and all the requisite
Petitioner Virginia A. Perez, on the other hand, supporting papers of the applicant. This happens
averred that the deceased had fulfilled all his only when it gives a policy.
prestations under the contract and all the elements
of a valid contract are present. It is not disputed, however, that when Primitivo
died on November 25, 1987, his application papers
On October 25, 1991, the trial court rendered a for additional insurance coverage were still with
decision in favor of petitioner ordering respondent the branch office of respondent corporation in
to pay 150,000 pesos. The Court of Appeals, Quezon. Consequently, there was absolutely no
however, reversed the decision of the trial court way the acceptance of the application could have
saying that the insurance contract for P50,000.00 been communicated to the applicant for the latter
could not have been perfected since at the time to accept inasmuch as the applicant at the time
that the policy was issued, Primitivo was already was already dead.
dead.
Petitioner insists that the condition imposed by BF
Petitioners motion for reconsideration having that a policy must have been delivered to and
been denied by respondent court, the instant accepted by the proposed insured in good health is
petition for certiorari was filed on the ground that potestative, being dependent upon the will of the
there was a consummated contract of insurance corporation and is therefore void. The court didnt
between the deceased and BF Lifeman Insurance agree. A potestative condition depends upon the
Corporation. exclusive will of one of the parties and is
considered void. The Civil Code states: When the
Issue: WON the widow can receive the proceeds of fulfillment of the condition depends upon the sole
the 2nd insurance policy will of the debtor, the conditional obligation shall
be void.
Held: No. Petition dismissed.
The following conditions were imposed by the
Perezs application was subject to the acceptance respondent company for the perfection of the
of private respondent BF Lifeman Insurance contract of insurance: a policy must have been
Corporation. The perfection of the contract of issued, the premiums paid, and the policy must
insurance between the deceased and respondent have been delivered to and accepted by the
applicant while he is in good health.
The car was then insured with State Insurance
The third condition isnt potestative, because the Company and the policy delivered to Mora.
health of the applicant at the time of the delivery
of the policy is beyond the control or will of the During the effectivity of the insurance contract, the
insurance company. Rather, the condition is a car figured in an accident. The company then
suspensive one whereby the acquisition of rights assigned the accident to an insurance appraiser for
depends upon the happening of an event which investigation and appraisal of the damage.
constitutes the condition. In this case, the
suspensive condition was the policy must have Mora without the knowledge and consent of HS
been delivered and accepted by the applicant while Reyes, authorized Bonifacio Bros to fix the car,
he is in good health. There was non-fulfillment of using materials supplied by the Ayala Auto Parts
the condition, because the applicant was already Company.
dead at the time the policy was issued.
For the cost of Labor and materials, Mora was
As stated above, a contract of insurance, like other billed P2,102.73. The bill was sent to the insurers
contracts, must be assented to by both parties appraiser. The insurance company drew a check in
either in person or by their agents. So long as an the amount of the insurance proceeds and
application for insurance has not been either entrusted the check to its appraiser for delivery to
accepted or rejected, it is merely an offer or the proper party.
proposal to make a contract. The contract, to be
binding from the date of application, must have The car was delivered to Mora without the consent
been a completed contract. of HS Reyes, and without payment to Bonifacio
Bros and Ayala.
The insurance company wasnt negligent because
delay in acting on the application does not Upon the theory that the insurance proceeds
constitute acceptance even after payment. The should be directly paid to them, Bonifacio and
corporation may not be penalized for the delay in Ayala filed a complaint against Mora and the
the processing of the application papers due to the insurer with the municipal court for the collection
fact that process in a week wasnt the usual of P2,102.73.
timeframe in fixing the application. Delay could not
be deemed unreasonable so as to constitute gross The insurance company filed its answer with a
negligence. counterclaim for interpleader, requiring Bonifacio
and HS Reyes to interplead in order to determine
who has a better right to the proceeds.
Bonifacio Bros. v. Mora
G.R. NO. L-20853, May 29, 1967 Issue: Whether or not there is privity of contract
between Bonficacio and Ayala on one hand and
Facts: Enrique Mora mortgaged his Odlsmobile State Insurance on the other.
sedan car to HS Reyes Inc. with the condition that
Mora would insure the car with HS Reyes as Held: NO. It is fundamental that contracts take
beneficiary. effect only between the parties thereto, except in
some specific instance provided by law where the
contract contains some stipulation in favor of a third persons have no right either in a court of
third person. Such stipulation is known as a equity, or in a court of law, to the proceeds of it,
stipulation pour autrui; or a provision in favor of a unless there be some contract of trust, expressed
third person not a party to the contract. or implied, by the insured and third person. In this
case, no contract of trust, express or implied. In
Under this doctrine, a third person is ed to avail this case, no contract of trust, expressed or implied
himself of a benefit granted to him by the terms of exists. We, therefore, agree with the trial court
the contract, provided that the contracting parties that no cause of action exists in favor of the
have clearly and deliberately conferred a favor appellants in so far as the proceeds of insurance
upon such person. Consequently, a third person are concerned. The appellant's claim, if at all, is
NOT a party to the contract has NO action against merely equitable in nature and must be made
the aprties thereto, and cannot generally demand effective through Enrique Mora who entered into a
the enforcement of the same. contract with the Bonifacio Bros Inc. This
conclusion is deducible not only from the principle
The question of whether a third person has an governing the operation and effect of insurance
enforceable interest in a contract must be settled contracts in general, but is clearly covered by the
by determining whether the contracting parties express provisions of section 50 of the Insurance
intended to tender him such an interest by Act (now Sec. 53).
deliberately inserting terms in their agreement
with the avowed purpose of conferring favor upon The policy in question has been so framed that
such third person. IN this connection, this court "Loss, if any, is payable to H. S. Reyes, Inc." which
has laid down the rule that the fairest test to unmistakably shows the intention of the parties.
determine whether the interest of a 3rd person in a
contract is a stipulation pour autrui or merely an
incidental interest, is to rely upon the intention of Coquia v. Fieldmens Insurance
the parties as disclosed by their contract. G.R. NO. L-23276, November 29, 1968

Facts: On Dec. 1, 1961, Fieldmens Insurance co.


In the instant case the insurance contract does not Issued in favor of the Manila Yellow Taxicab a
contain any words or clauses to disclose an intent common carrier insurance policy with a stipulation
to give any benefit to any repairmen or material that the company shall indemnify the insured of
men in case of repair of the car in question. The the sums which the latter wmy be held liable for
parties to the insurance contract omitted such with respect to death or bodily injury to any faire-
stipulation, which is a circumstance that supports paying passenger including the driver and
the said conclusion. On the other hand, the "loss conductor.
payable" clause of the insurance policy stipulates
that "Loss, if any, is payable to H.S. Reyes, Inc." The policy also stated that in the event of the
indicating that it was only the H.S. Reyes, Inc. death of the driver, the Company shall indemnify
which they intended to benefit. his personal representatives and at the Companys
option may make indemnity payable directly to the
A policy of insurance is a distinct and independent claimants or heirs of the claimants.
contract between the insured and insurer, and
During the policys lifetime, a taxicab of the insured that they could and did properly join the latter in
driven by Coquia met an accident and Coquia died. filing the complaint herein.

When the company refused to pay the only heirs of


Coquia, his parents, they institued this complaint.
The company contends that plaintiffs have no
cause of action since the Coquias have no
contractual relationship with the company.

Issue: Whether or not plaintiffs have the right to


collect on the policy.

Held: YES. Athough, in general, only parties to a


contract may bring an action based thereon, this
rule is subject to exceptions, one of which is found
in the second paragraph of Article 1311 of the Civil
Code of the Philippines, reading: "If a contract
should contain some stipulation in favor of a third
person, he may demand its fulfillment provided he
communicated his acceptance to the obligor before
its revocation. A mere incidental benefit or interest
of a person is not sufficient. The contracting parties
must have clearly and deliberately conferred a
favor upon a third person." This is but the
restatement of a well-known principle concerning
contracts pour autrui, the enforcement of which
may be demanded by a third party for whose
benefit it was made, although not a party to the
contract, before the stipulation in his favor has
been revoked by the contracting parties

In the case at bar, the policy under consideration is


typical of contracts pour autrui this character being
made more manifest by the fact that the deceased
driver paid fifty percent (50%) of the corresponding
premiums, which were deducted from his weekly
commissions. Under these conditions, it is clear
that the Coquias who, admittedly, are the sole
heirs of the deceased have a direct cause of
action against the Company, and, since they could
have maintained this action by themselves, without
the assistance of the insured it goes without saying

You might also like